LSAT and Law School Admissions Forum

Get expert LSAT preparation and law school admissions advice from PowerScore Test Preparation.

User avatar
 Dave Killoran
PowerScore Staff
  • PowerScore Staff
  • Posts: 5852
  • Joined: Mar 25, 2011
|
#27109
Complete Question Explanation
(The complete setup for this game can be found here: lsat/viewtopic.php?t=49)

The correct answer choice is (C)

The condition in the question stem suspends the second rule. With the second rule suspended, only three rules remain in effect: the first, third, and fourth rules. As none of the answer choices contain K, the first rule is inapplicable. Two of the answer choices—(C) and (E)—contain O, but both of those answer choices also contain P and W, so there is no violation of the third rule. Two answers—(C) and (E)—contain W, but while (E) also contains F, (C) contains neither F nor T, and thus answer choice (C) violates the fourth rule and is therefore the correct answer.

Get the most out of your LSAT Prep Plus subscription.

Analyze and track your performance with our Testing and Analytics Package.